water001 发表于 2005-8-5 20:11

求教一个组合数学问题

组合数学中,有这样一个定理:
Frobenius Th: 设 、 为正整数,且 ,则大于 的任意整数 都可以写作 的形式,其中 、 均为非负整数,即方程 至少有一组非负整数解。
我的问题是:(1)当 、 、 为正整数,且两两互素时,类似于Frobenius Th的结论的正整数 的下界是多少?进一步,当 、 为正整数,且两两互素时,有没有类似的结论?关于这方面的问题,可参阅哪些文献资料?
(2)在FrobeniusTh中,去掉 这一条件,原来的结论是否成立?我个人认为是成立的!
(3)设 、 为正整数,且 ,小于的正整数有个可表示的非负线性组合,这一结论我在一本书中看见过,用发生函数证明的,有没有更简捷的证法?这一结论能否象(1)那样推广到多于两个正整数的情形?
页: [1]
查看完整版本: 求教一个组合数学问题